You are on page 1of 19

CICLO PREUNIVERSITARIO ADMISIN -II SEMINARIO N 02

CEPRE-UNI FSICA 1
FSICA

01. La ... es aquella capaz de
cambiar el estado de movimiento de
una partcula, y su expresin
dimensional es
Completan la oracin:
A) masa ; M
B) aceleracin; LT
2

C) fuerza ; MLT
2

D) magia ; MLT
E) imaginacin; *

02. Con relacin a las siguientes
proposiciones sobre la primera Ley de
Newton indique verdadero (V) o falso
(F):
I. Una partcula que se encuentra
viajando con velocidad constante
continuar en dicho estado de
movimiento, salvo que una fuerza
actu sobre l.
II. Un cuerpo que est en reposo
puede iniciar su movimiento
repentinamente sin la accin de
una fuerza.
III. Para que un cuerpo tenga un
movimiento rectilneo uniforme, es
necesario la accin de una fuerza
constante.

A) VVV B) VVF C) VFV
D) FVV E) VFF

03. Respecto a la 1 Ley de Newton
seale la veracidad (V) o falsedad (F)
de las siguientes proposiciones.
I. La primera ley de Newton es
aplicable tambin en un sistema de
referencia acelerado respecto a
Tierra.
II. En un reposo instantneo respecto
a Tierra es aplicable la 1 Ley de
Newton.
III. Los objetos en reposo o con MRU
respecto a Tierra cumplen con la
1 Ley de Newton.
A) VFV B) VVF C) FFV
D) FFF E) FVF

04. Se propone lo siguiente:
I. La expresin dimensional MLT
2

tiene unidad newton en el SI.
II. La 1era Ley de Newton explica
porqu al romperse la cuerda que
ata a una piedra en movimiento
circular, sta se mueve en una
recta tangente a la trayectoria que
tena.
III. Si un objeto se mueve con MRU
respecto de otro, entonces segn
la 1era Ley de Newton existe
equivalencia entre ambos, es decir
est en equilibrio uno respecto al
otro.
Son correctas:

A) I B) II C) III
D) I y II E) II y III

05. Respecto a la 1era Ley de Newton,
determinar si las proposiciones
siguientes son falsas (F) o verdaderas
(V).
I. Si la fuerza sobre una partcula es
nula, sta necesariamente tiene un
MRU.
II. Si la fuerza sobre una partcula es
nula, sta necesariamente est en
reposo.
III. Si no existe accin externa sobre
una partcula, sta necesariamente
est en reposo o con MRU.

A) VVV B) FFF C) FFV
D) FVV E) VFF

06. Indique verdadero (V) o falso (F),
segn corresponda:
I. Cuando se lanza una moneda
hacia arriba, en el punto ms alto
de su trayectoria, se encuentra en
equilibrio.
II. Un tren se mueve con velocidad
constante igual a
0 V

. Si una
CICLO PREUNIVERSITARIO ADMISIN -II SEMINARIO N 02

CEPRE-UNI FSICA 2
Cuerda
A
B
F
x
y
0
manzana se encuentra en reposo
sobre una mesa del tren entonces
sobre la manzana debe actuar una
fuerza paralela a
0 V

.
III. La interaccin entre un cuerpo y la
superficie que le sirve de apoyo es
de naturaleza electromagntica.

A) VVV B) VVF C) VFF
D) VFV E) FFV

07. Indicar la veracidad (V) o falsedad (F)
de las proposiciones siguientes:
I. Las fuerzas bsicas en la
naturaleza son cinco.
II. Las fuerzas de interaccin
electrodbil tienen menor alcance
que las de interaccin gravitatoria.
III. El cambio en el estado de
movimiento de los electrones en
torno del ncleo de un tomo, es el
resultado de la interaccin nuclear
fuerte.

A) VVV B) FFF C) FFV
D) FVF E) VVF

08. Seale verdadero (V) o falso (F) las
siguientes proposiciones con respecto
a la 3ra Ley de Newton.
I. Es consecuencia de la primera ley
de Newton.
II. Las fuerzas se presentan en pares.
III. Es vlida solamente usando los
cuerpos estn adyacentes.

A) VVV B) VVF C) VFF
D) FVF E) FFF

09. Las esferas A y B estn apoyadas
sobre superficies lisas tal como lo
indica la figura. Indique el D.C.L. de la
esfera A, si sobre B acta una fuerza
vertical
( )
F F j




















A) B) C)








D) E)







10. La figura muestra a Pedro (peso P)
sentado en reposo sobre un andamio
(peso p), tirando de una cuerda ideal
(que pasa por la polea, tambin ideal)
que soporta una tensin T. cul es el
DCL correcto?











CICLO PREUNIVERSITARIO ADMISIN -II SEMINARIO N 02

CEPRE-UNI FSICA 3
P
p
T
T
P
tp
A) B)







C) D)








E)







11. La barra que se muestra es
homognea y se mantiene en
equilibrio apoyada en una pared
vertical lisa y en un piso horizontal
rugoso como se indica en el dibujo (A).
es correcto que:
I. Al descomponer la reaccin del
piso sobre la barra. Su diagrama
de cuerpo libre queda como indica
la figura (B).
II. El peso y la reaccin normal del
piso forman un par de accin-
reaccin.
III. La reaccin de la pared y el
rozamiento forman un par de
accin-reaccin.

















A) Todas B) Solo I C) I y II
D) I y III E) Ninguna

12. Dos cilindros macizos de y 15 kg
de masa, se apoyan sin rozamiento
como se muestra en la figura. Calcule
el ngulo que forma con la horizontal
la lnea que une los centros de los
cilindros.








A) arctg(1/4) B) arctg(1/2)
C) 30 D) 45
E) 60

13. En la figura halle el valor de la normal
N en funcin de W considerando que
el sistema est en equilibrio.








A) W B) W 2/3
C) W 3/3 D) 2W
E) 3W
T
P
2T
P
2T
P +p

37
53
liso
?
peso
Reaccin de
la pared
Reaccin
normal del
piso
Rozamiento
A
B
T


W
30
30
0 =
CICLO PREUNIVERSITARIO ADMISIN -II SEMINARIO N 02

CEPRE-UNI FSICA 4
14. En la figura las masas estn en
equilibrio. Seale la veracidad (V) o
falsedad (F) de las siguientes
proposiciones:
I.
m
1
m'
<
II. La normal N sobre m' en funcin
de vara aproximadamente
segn la figura.





III.
m'
1
m
<







A) FFF B) VVF C) VFF
D) FVF E) FVV

15. Dos cilindros de radios
1
R 4m = y
2
R m = y masas
1
m 10kg = y
2
m 2kg = respectivamente se
encuentran en equilibrio. Halle la
constante K en N/m del resorte si ste
se deforma 8 cm.








A) 150 B) 300 C) 450
D) 600 E) 900




16. Una barra homognea de N de
peso y 10 m de longitud puede girar
alrededor del punto O, si es
abandonada en la posicin A.
Determine el torque (en N.m) debido al
peso, en el instante que la barra pasa
por B, con respecto del centro de
rotacin.










A) 20 3k
$
B) 50 3k
$

C) 20 3k -
$
D) 25 3k -
$

E) 50 3k -
$


17. Sobre una placa actan tres fuerzas
como se muestran cuyos mdulos son:
1
F 60 N = ,
2
F 50 N = ,
3
F 10 N = . Si P
es punto medio, determine el torque
resultante (en N.m) respecto al punto
P.









A)

34 k B)

38 k C)

26 k
D)

42 k E)

32 k






N

37
60
B
A
y

x

z
g


O

2
F


1
F


4 m
P
53

3 m
3
F


0 =
m'
m

CICLO PREUNIVERSITARIO ADMISIN -II SEMINARIO N 02

CEPRE-UNI FSICA 5
18. Determine el torque (en N m ) de la
fuerza F

respecto del punto A.












A)

148,5i 124j 248,6k +




B)

184,6i 248j 424,8k +




C)

151,5i 202j 363,6k + +




D)

38,5i 145,6j 228,6k +




E)

148,6i 264,8j 326,8k +





19. En la figura calcule la fuerza de
friccin. Considere que la barra es
homognea y se encuentra en
equilibrio.










A) 25 B) 25 3 C) 50
D) 50 E) 50 3

20. Una bolita de peso P parte de A y se
mueve hacia B con una velocidad
constante de cm/s, sobre una viga
horizontal. Considerando el peso de la
viga despreciable, determine el tiempo
en segundos en el cual la reaccin en
el soporte A es el doble de la reaccin
en B.








A) 80 B) 100 C) 70
D) 60 E) 110

21. En la figura la viga de 1 m con una
masa de 6 kg est apoyada en un piso
rugoso y en una esfera lisa como se
indica. Si la esfera de radio a tiene una
masa de 10 kg. Determine (en N) la
reaccin normal del piso sobre la viga
cuando sta se encuentra a punto de
deslizar.








A) 25 B) 30 C) 40
D) 45 E) 50

22. La barra homognea de N de
peso, se mantiene en reposo en la
posicin mostrada, halle la tensin en
la cuerda (en N).

A) 4
B) 5
C) 6
D) 8
E) 10









30

30
T

W =200 N



37
v

B A
3 m
z

x(m)

y(m)

5

B

100 N

(8,3,0)

C

A

(4,6,0)

F


45 =
s
0,5 =
CICLO PREUNIVERSITARIO ADMISIN -II SEMINARIO N 02

CEPRE-UNI FSICA 6
23. En la figura se muestran 2 esferas del
mismo material cuyos radios son
a cm = y b 2 cm = apoyadas en
una superficie hemisfrica de radio
11 cm si ignoramos la friccin y
1
sen
6
= , determine que define la
posicin de equilibrio.







A) 15 B) 30 C) 37
D) 45 E) 22,5

24. Se dispone de una varilla de 3 m para
levantar un saco de kg. Si se
utiliza una fuerza de 100 N, cul es la
mxima distancia (en cm), medida
desde el extremo donde se coloca al
saco, donde debe ubicarse el apoyo a
la varilla?
A) 15 B) 27 C) 32
D) 45 E) 52

25. Determine la fuerza F necesaria para
mantener al bloque de peso W en
equilibrio, sobre el plano inclinado liso.
Todas las poleas son de peso
despreciable.













A)
2
W
5
B)
9
W
100
C)
3
W
25

D)
3
W
20
E)
3
W
100


26. El sistema es ideal, las poleas no tiene
pesos, no hay friccin, determine la
razn de masas
B A
m /m , para obtener
el equilibrio.









A) 3,0 B) 2,4 C) 1,8
D) 1,2 E) 0,4

27. Un bloque de kg se encuentra
sobre un plano inclinado 45, si la
fuerza de rozamiento entre el bloque y
el plano es insignificante, determine la
fuerza mnima horizontal (en N) capaz
de mantener al bloque en reposo
( )
2
g 10 m/s =
A) 500 B) 500 2
C) 1000 D) 1000 2
E) 750

28. Entre que valores, debe estar la fuerza
que la persona aplica a la caja
(paralela al plano inclinado) sin que
esta llegue a deslizar. (Peso de la caja
100 N;
s
3/6, 30 = = ).






A) 20 y 50 B) 30 y 90 C) 25 y 75
D) 25 y 80 E) 30 y 75
37




F




g





37




y




x





R


a
b
B
37
CICLO PREUNIVERSITARIO ADMISIN -II SEMINARIO N 02

CEPRE-UNI FSICA 7
29. Un bloque de kg de masa est
apoyado sobre una superficie
horizontal spera, al tomar mediciones
de la fuerza horizontal F y la fuerza de
rozamiento se obtiene la siguiente
grfica. Indique las afirmaciones
verdaderas.











I. El coeficiente de rozamiento
esttico entre las superficies es
0,75.
II. El coeficiente de rozamiento por
deslizamiento es 0,5.
III. La fuerza de rozamiento por
deslizamiento es 800 N.

A) Solo I B) Solo II C) Solo III
D) Solo I y II E) Solo II y III

30. Del grfico, al aplicar una fuerza
F = 60 i

N, el bloque de kg de
masa, se encuentra en estado de
movimiento inminente hacia arriba; si
la esfera tiene una masa de 5 kg,
determine el coeficiente de rozamiento
esttico entre el bloque y la
pared ( )
2
g 10 m/s = .








A) 0,5 B) 0,4 C) 0,3
D) 0,2 E) 0,6
31. En la figura la esfera en equilibrio est
apoyada en una rampa rugosa.
Calcule la fuerza de friccin si la
prolongacin de la cuerda pasa por el
centro de dicha esfera.







A) W cos B) W sen
C) 0 D) Wtan
E) W ctan

32. El muchacho desea poner en
movimiento un bloque cuyo peso es 4
veces el suyo. Halle el mnimo
coeficiente de rozamiento que debe
existir entre sus zapatos y el piso.
Entre el bloque y el piso:
s
0,12 = ;
k
0,10 = .




A) 0,03 B) 0,06 C) 0,12
D) 0,40 E) 0,48

33. La figura muestra un bloque de
15 3kg, al cual se le aplica una
fuerza horizontal de N, haciendo
que el bloque se mueva con velocidad
constante a lo largo del plano,
determine la magnitud (en N) de la
reaccin del plano inclinado sobre el
bloque. ( )
2
g 10 m/s = .





A) 150 B) 150 3 C) 300
D) 300 3 E) 450
m F
200
f(N)
0
400
600
F(N)
200 400 600 800

C
W
F
37
F
CICLO PREUNIVERSITARIO ADMISIN -II SEMINARIO N 02

CEPRE-UNI FSICA 8
34. Mediante una fuerza horizontal se
desea llevar un bloque de N hacia
arriba, sobre el plano inclinado, con
movimiento uniforme. Si el coeficiente
de friccin cintico entre el bloque y el
plano es 0,5. Determine la magnitud
de dicha fuerza (en N) ( )
2
g 10 m/s =







A) 175 B) 200 C) 225
D) 250 E) 275

35. En el sistema mostrado en la figura las
partculas tiene masas iguales y estn
en equilibrio. Halle el valor mnimo de
u para que no se deslicen.
1 2
m m =







A) 0,2 B) 0,3 C) 0,5
D) 0,7 E) 0,8

36. Una escalera de longitud L y N
de peso, descansa en reposo como se
muestra en la figura. Halle la friccin
(en N) con el piso
( )
53 = .

A) 20
B) 30
C) 40
D) 45
E) 60




37. La escalera de 5 m de longitud tiene
una masa de 20 kg, un nio de 40 kg
sube por ella, si solo hay friccin en la
superficie horizontal ( )
s k
0,5 y 0,4 = = .
Determine la altura h que puede subir
sin que el conjunto deslice (en m).









A) 1 B) 2 C) 3
D) 4 E) 5

38. La barra de 8 kg mostrada en la figura,
se apoya en un plano inclinado liso
(punto A) y la articulacin (punto B). El
coeficiente de friccin en el plano
horizontal es
s
0,6 = ,
k
0,5 = . Si el
sistema est a punto de deslizarse,
determine la reaccin de la superficie
sobre la barra en el punto A (en N)
considere la pesa de 1 kg pequea.







A) 40 B) 44 C) 50
D) 90 E) 100

39. Si la grfica representa la aceleracin
que adquiere un bloque al aplicrsele
una fuerza, seale verdadero (V) o
falso (F):
I. El grfico es una consecuencia de
la 2 ley de Newton.
II.
1 2
m m > .
III.
2 1
m m >

37 =

F
53
53
h
B
A
37
A
1 kg
B
s
0
liso
CICLO PREUNIVERSITARIO ADMISIN -II SEMINARIO N 02

CEPRE-UNI FSICA 9








A) VVV B) VVF C) VFV
D) VFF E) FFF

40. Un cuerpo de kg de masa se
desplaza sobre una superficie
horizontal rugosa (
k
0,2 = ) bajo la
accin de una fuerza F paralela a la
superficie. Si la velocidad del cuerpo
vara con el tiempo de acuerdo a la
grfica que se muestra, determine la
magnitud (en N) de la fuerza F.
( )
2
g 10 m/s =







A) 12,5 B) 15,0 C) 20,0
D) 25,0 E) 27,5

41. Si el bloque C es abandonado en la
posicin mostrada. Determine la
aceleracin que adquiere el bloque A
(en m/s
2
). Desprecie el rozamiento en
todas las superficies en contacto.
( )
2
g 10 m/s = , si
A C B
m 3m 6m 12m = = =









A) 0,2 B) 1,2 C) 2,2
D) 3,2 E) 4,2

42. Se observa que cuando se aplica una
fuerza de F N = a un bloque de
10 kg su aceleracin resulta ser la
tercera parte de la que tiene cuando la
fuerza que se aplica es F 80 N = .
Determine el coeficiente de friccin
cintica.




A) 0,20 B) 0,25 C) 0,30
D) 0,35 E) 0,40

43. Indique las afirmaciones correctas
respecto de una partcula con
movimiento circular.
I. Si mantiene su velocidad angular
constante la fuerza resultante sobre
la partcula es nula.
II. Una partcula con aceleracin
angular constante cuyo radio de
trayectoria es R experimenta una
fuerza tangencial nula.
III. La velocidad lineal de la partcula
cambia de direccin debido a la
fuerza centrpeta.
A) Solo I B) Solo II C) Solo III
D) Solo I y II E) Solo II y III

44. Cul es el DCL que mejor representa
a la partcula en el punto A.











a
m
2

m
1

F
v (m/s)
4 2
t (s)
0
2
4
B

C

A

g



F
liso
A
g


CICLO PREUNIVERSITARIO ADMISIN -II SEMINARIO N 02

CEPRE-UNI FSICA 10

A) B) C)





D) E)





45. La figura muestra una partcula de
masa m, atada a un hilo, que realiza
un movimiento circular en un plano
vertical. Seale la veracidad o falsedad
(F) de las siguientes proposiciones:
I. En A la fuerza centrpeta es mayor
que la tensin.
II. En B la mnima fuerza centrpeta es
la tensin.
III. En C la fuerza centrpeta es menor
que la tensin.










A) FFV B) VVV C) VVF
D) VFV E) FFF

46. Sobre una pista circular de 450 m de
radio, un auto de 1000 kg acelera
desde el reposo hasta adquirir luego
de 9 s una rapidez de 30 m/s. Halle la
relacin entre los componentes
centrpeta y tangencial de la fuerza
que el piso ejerce sobre el auto.
A) 0,2 B) 0,4 C) 0,6
D) 0,8 E) 1,0

47. Un objeto de kg se mueve en una
trayectoria circular en una cuerda de
0,5 m de largo. Si se mantiene una
rapidez constante de 4 m/s, cul es la
tensin en N en la cuerda cuando el
objeto est en la posicin mostrada en
la figura? Es el movimiento del tipo
MCUV?











A) 15,8 ; no B) 39,5 ; si
C) 39,5 ; no D) 15,2 ; no
E) 20,7 ; si

48. Una partcula de masa m 2 kg =
describe un M.C.U.V con una
trayectoria horizontal de radio R 1m = .
En el punto A de la trayectoria la
fuerza sobre la partcula es
F 8 2iN =

, calcule aproximadamente
la velocidad en el punto B de la
trayectoria.








A) 2 B) 4 C) 6
D) 8 E) 10





N


c
F


c
F


c
F


45
A
B
45
37
g


y
x
C
B
m
cuerda
g


A
CICLO PREUNIVERSITARIO ADMISIN -II SEMINARIO N 02

CEPRE-UNI FSICA 11
49. Un piloto desciende en picada con su
avin y cuando su velocidad es de
700 km/h describe una trayectoria
semicircular en un plano vertical,
manteniendo su rapidez constante. Si
sabe que puede soportar en el punto
ms bajo de la trayectoria un peso
aparente de hasta 6 veces su peso, el
menor radio posible de esta trayectoria
semicircular debe medir (en m)
aproximadamente:
( )
2
g 9,8 m/s =
A) 756 B) 759 C) 763
D) 768 E) 771

50. Sobre un riel en forma de
semicircunferencia de m de radio
que se encuentra en un plano vertical
puede deslizarse una partcula de
masa m. Hasta que altura h (en m)
subir la masa cuando el riel gira con
una velocidad angular de 5 rad/s, tal
como se muestra en la figura.











A) 0,2 B) 0,5 C) 1,2
D) 1,6 E) 1,9

51. Respecto a los sistemas de referencia
inerciales, indique la veracidad (V) o
falsedad (F) de las proposiciones
siguientes:
I. Un sistema de referencia es
inercial, solo si est en reposo
respecto a Tierra.
II. Las leyes de Newton solo se
cumplen en los sistemas de
referencia inerciales.
III. Un cuerpo solo puede estar en
reposo o con MRU, respecto a un
sistema de referencia inercial.
A) VVV B) FFF C) VFV
D) VVF E) FVF


52. Un vagn con aceleracin constante
a se desplaza hacia la derecha, en
su interior un pasajero suelta una
moneda. Qu alternativa representa
mejor la trayectoria de la moneda
respecto al pasajero?











A) B) C) D) E)

53. Dadas las siguientes proposiciones:
I. En un sistema de referencia inercial
todos los cuerpos sometidos a
resultante de fuerzas cero o se
encuentran en reposo o
movindose con velocidad
constante.
II. Un sistema de referencia inercial es
aquel que cumple con la 1ra Ley de
Newton.
III. Desde un mvil que se encuentra
en Tierra, con velocidad constante,
respecto a un observador se
observa un objeto que se mueve
con velocidad constante, entonces
sobre dicho objeto la suma de
fuerzas es cero.
Cules son verdaderas?
A) Solo I B) Solo III C) Todas
D) I y II E) II y III

=5 rad/s
h
superficie
lisa
a


CICLO PREUNIVERSITARIO ADMISIN -II SEMINARIO N 02

CEPRE-UNI FSICA 12
54. Con relacin al concepto de sistemas
de referencia inerciales (SRI), seale
la veracidad (V) o falsedad (F) de las
siguientes proposiciones:
I. La aceleracin de un cuerpo es la
misma en cualquier SRI.
II. La velocidad de un cuerpo es la
misma en cualquier SRI.
III. Si un cuerpo no est acelerado
respecto a un sistema de
referencia, entonces dicho sistema
es un SRI.
A) FFF B) VFV C) VFF
D) FVV E) FVF

55. Dos observadores A y B uno fijo a
Tierra (A) y otro que se mueve con
velocidad constante V

(B), observan
que una esferita C es abandonada,
con relacin a las siguientes
proposiciones indique verdadero (V) o
falso (F).
I. Solo A es considerado un sistema
de referencia inercial.
II. Los dos observadores medirn el
mismo tiempo de cada de C.
III. La trayectoria de C observada por
ambos sistemas (A) y (B) son
rectilneas.








A) FVF B) VVF C) FFF
D) FVV E) VFF

56. El grfico muestra un satlite AB en
rbita alrededor de un planeta; si el
satlite recorre el tramo AB en
das y barre un rea que es el 30% del
rea COD; calcule el periodo (en das)
del satlite.









A) 10 B) 15 C) 30
D) 20 E) 25

57. El dibujo muestra un planeta con los
satlites A y B. Si el periodo del
satlite A es de h, calcule el
periodo de B. Asuma que
B A
R 1,5 R = .









A) 32,8 B) 43,2 C) 45,9
D) 49,4 E) 53,2

58. Un satlite artificial m es puesto en
rbita alrededor de la Tierra con un
periodo de rbita de 2,8 das, un radio
de rbita 5a (en km). Determine
aproximadamente el periodo (en das)
de otro satlite artificial n cuyo radio
de rbita es 4a (en km).
A) 0,5 B) 2 C) 3
D) 4 E) 5

59. El cometa Halley se acerca al Sol a
una distancia aproximada de 0,57 UA,
y su periodo orbital es de aos.
Qu tan lejos del sol viajar el
cometa Halley antes de que inicie su
viaje de regreso? (UA: unidad
astronmica
8
1UA 1,5 10 km = es la
distancia media Tierra-Sol).
A
B
Tierra
C
V


D
B
o
A
C
B
R
A
R
A
B
CICLO PREUNIVERSITARIO ADMISIN -II SEMINARIO N 02

CEPRE-UNI FSICA 13
30
sol
M 1,99 10 kg = . Dar la respuesta
en U.A.
A) 5 B) 12 C) 17
D) 21 E) 35

60. Dos satlites artificial y natural orbitan
alrededor de un planeta, tal como se
muestra, determine el mximo
alejamiento del satlite (2) respecto del
planeta, si su periodo es 5 5 veces el
periodo del satlite (1).








A) 5R B) 7R C) 9R
D) 10R E) 15R

61. Con respecto a las siguientes
proposiciones, seale (V) o falso (F):
I. La relacin
2
3
T
cte
R
= se cumple
para cualquier par de planetas
independientemente del sistema al
cual pertenezcan (la cte es la
misma).
II. La 3 ley de Kepler (ley del
cuadrado del periodo) se
demuestra aplicando la 2 ley de
Newton.
III. El radio medio usado en la
aproximacin circular de la
trayectoria elptica de un planeta,
es la semisuma de los semiejes
mayor y menor de la elipse.

A) VVV B) VFV C) VVF
D) FVF E) FFF




62. Sea F la fuerza gravitacional entre los
planetas M y 3M. Halle la magnitud de
la fuerza gravitacional sobre el cometa
m, en la posicin mostrada, debido a
M y 3M.









A)
m
3 F
M



B)
m
13 F
M




C)
M
13 F
m



D)
1 m
F
3 M




E)
13 m
F
3 M





63. Halle el valor de la masa m' para que
la fuerza de gravitacin resultante
sobre la masa m, ms prxima a m',
sea cero. El polgono es un hexgono
regular.









A) m/4 B) m/2 C) 3m/4
D) m E) 5m/4

64. Los radios de las rbitas circulares
alrededor del Sol de Saturno y Urano
son
12
1,43 10 m y
12
2,87 10 m
respectivamente. Encuentre la relacin
entre las aceleraciones de los
planetas.
A) 4 B) 8 C) 16
D) 24 E) 32
m
m
m
m
m' ? =
a
a
a
x
(1)
(2)
PLANETA
R
3M M
60 60
m
CICLO PREUNIVERSITARIO ADMISIN -II SEMINARIO N 02

CEPRE-UNI FSICA 14
65. Para el sistema Sol-Tierra-Luna,
determine el cociente entre las
magnitudes de las fuerzas mxima y
mnima, actuantes sobre la Tierra, en
trminos de
Sol
Luna
M
x
M
= y de
Sol Tierra
Tierra Luna
R
y
R

= . Suponga rbitas
circulares.
A)
2
2
x y
x y
+

B)
2
2
x y
x y

+

C)
2
2
x y
x y
+

D)
2
2
x y
x y

+

E)
x y
x y
+



66. Un cuerpo pesa N sobre la
superficie de un supuesto planeta de
radio 4100 km. Determine a que altura
(en km) sobre la superficie su peso
ser 3,96% menor.
A) 600 B) 500 C) 400
D) 350 E) 83,7

67. Determine a que altura (en km) sobre
la superficie de la Tierra un cuerpo
tendr la mitad de su peso (considere
T
R 6400 km = ).
A) 1381 B) 2105 C) 2624
D) 2942 E) 3557

68. A qu altura sobre la superficie
terrestre aproximadamente la
aceleracin de la gravedad es el 90%
de su valor al nivel del mar? (R, radio
de la Tierra)
A) R/10 B) R/20 C) R/30
D) R/40 E) R/50

69. Se sabe que g es la aceleracin de la
gravedad en la superficie de un
planeta. Determine el valor de la
aceleracin de la gravedad a una
altura igual a 2,5R donde R es el radio
del planeta.
A) g/49 B) 4g/49 C) 5g/49
D) 6g/49 E) 7g/49

70. A qu altura sobre la superficie (en
km) de la Tierra la aceleracin de la
gravedad se reduce a la mitad del
valor en la superficie? En qu
porcentaje se reduce g a km de
altura sobre la superficie terrestre?
2
g 9,8 m/s = en la superficie de la
Tierra, (
24
T
M 6 10 = kg,
T
R 6400 km =
A) 9030 ; 2% B) 2630 ; 4%
C) 15500 ; 2% D) 2630 ; 8%
E) 9030 ; 4%

71. A qu altura sobre la superficie de la
Tierra (en trminos de su radio R) el
valor de la aceleracin de la gravedad
es la cuarta parte del valor que le
corresponde en su superficie?
A)
R
2
B) R C) 2R
D)
5
R
2
E) 3R

72. Si el bloque de kg sube 4 metros
con velocidad constante, halle el
trabajo de la fuerza F (en J ) g 10 =
m/s
2
.








A) 200 B) 200 C) 100
D) 400 E) 800





F
CICLO PREUNIVERSITARIO ADMISIN -II SEMINARIO N 02

CEPRE-UNI FSICA 15
73. El sistema mostrado empieza a
moverse a partir del reposo. Halle el
trabajo que realiza la tensin de la
cuerda, para desplazar al bloque de
5 kg una longitud de 7 m.
( )
2
g 10 m/s =








A) 100 B) 200 C) 100
D) 400 E) 800

74. Sobre el objeto (m kg) =
mostrado en la figura, acta una
fuerza F 2i N =

. Si el objeto se
desplaza 3i m

, seale la veracidad (V)


o falsedad (F) de las siguientes
proposiciones:
I. El trabajo hecho por F es 6 J .
II. El trabajo hecho por la fuerza
resultante es 6 J .
III. El trabajo hecho por la fuerza de
friccin es 1 J .





A) VVV B) VFV C) VFF
D) FVV E) FFF

75. Sobre el bloque de masa m mostrado
en la figura, acta una fuerza F
(x)

paralela al piso de magnitud variable.
Halle el trabajo de F(x) desde x 0 =
hasta x d = .












A)
0
F d
2
B)
0
F d
C) 0 D)
0
F d
2

E)
0
F d

76. En el extremo inferior de un resorte de
20 cm de longitud natural se cuelga un
bloque de 5 kg, el resorte se estira
5 cm. Luego, muy lentamente, se
aplica al bloque una fuerza F vertical,
que lo hace descender 10 cm. Calcule
el trabajo (en J ) realizado por la fuerza.
( )
2
g 10 m/s =
A) 5 B) 8 C) 10
D) 12 E) 15

77. En la figura se muestra un bloque
sobre una superficie horizontal rugosa
inicialmente en reposo, si al bloque se
aplica una fuerza variable cuyo mdulo
es F 10x 20 = + , x (en m) y F (en N)
considerando
k
0,5 = , determine el
trabajo de la friccin (en J ) desde
x 0 = hasta x 6m = , considere la masa
del bloque 3 kg. ( )
2
g 10 m/s =







A) 90 B) 120 C) 150
D) 180 E) 240

O
X
F(x)
O
X
d/2
F
0

F
0

d
F
0,5 = 37
F
x
y
x
2 kg

5 kg

liso

g


m
F

k
=0,1

CICLO PREUNIVERSITARIO ADMISIN -II SEMINARIO N 02

CEPRE-UNI FSICA 16
78. Si se desprecia todo tipo de
rozamiento, qu trabajo (en J ) realiza
la fuerza no conservativa F vertical que
levanta la cadena de kg de masa y
cuya longitud es 4 m, inicialmente en
reposo como se muestra en la figura,
cuando es desplazado hasta el
instante en que el ltimo eslabn
abandona la superficie con una
velocidad de 10 m/s? ( )
2
g 10 m/s =






A) 1800 B) 1200 C) 500
D) 700 E) 450

79. Halle el trabajo neto (en J ) necesario
para trasladar un cuerpo de g de
masa, de un punto (A) a otro (B)
distantes entre si 6 m en un tiempo de
6 s. El cuerpo parte del reposo y el
movimiento es rectilneo
uniformemente variado.
A) 6,0 B) 10
6
C)
6
6 10
D) 0,6 E) 60

80. Un bote se est desplazando con una
energa cintica K. Qu trabajo debe
realizar el viento sobre las velas del
bote para que este duplique su
velocidad?
A) K B) 2K C) 3K
D) 4K E) 5K

81. Un bloque de kg, en reposo en el
origen de coordenadas, inicia su
movimiento bajo la accin de una
fuerza F 3x = (en newton). Cul es la
variacin (en J ) en su energa cintica
cuando se desplaza desde x 1 = hasta
x 3m = ?

A) 4 B) 8 C) 12
D) 16 E) 20

82. La figura muestra un bloque de
madera de longitud L, una bala lo
impacta con una velocidad inicial V
0
y
sale de mismo con una velocidad
0
v
v
2
= . Cul debe ser la longitud de
otro bloque, del mismo material que el
anterior, para que al impactarle una
bala con velocidad igual a v
0
se
detenga justo al salir?







A)
L
3
B) L C)
4
L
3

D)
3
L
2
E) 2L

83. Una partcula de 4 kg se somete a una
fuerza que vara con la posicin, como
se muestra en la figura. La partcula
parte con una velocidad v 2i =

m/s.
Cul es la velocidad (en m/s) en (a)
5 m y en (b) 15 m?









A) 1,7 ; 2,9 B) 2,9 ; 4,5
C) 2,2 ; 3,7 D) 3,4 ; 5,8
E) 4,5 ; 6,3



Fx(N)
x(m)
6
5 10 15
F
L
v
0

CICLO PREUNIVERSITARIO ADMISIN -II SEMINARIO N 02

CEPRE-UNI FSICA 17
84. Un cuerpo parte del reposo y se
mueve sobre una superficie horizontal
por efecto de una fuerza de magnitud
variable que acta siempre en la
misma direccin y sentido, tal como se
muestra en la figura. Si la masa del
cuerpo es de kg. Cul es su
rapidez (en m/s) cuando x 1,2m = ?













A) 0,94 B) 1,82 C) 2,03
D) 2,77 E) 3,01

85. Una partcula puede desplazarse a lo
largo de cualquiera de las trayectorias
que se muestra en la figura. La fuerza
actuante sobre la partcula es
conservativa a lo largo de todas las
trayectorias excepto a lo largo de la
trayectoria CE. Si los trabajos
ejecutados por la fuerza son
AD DE BC
W W W 0 = = = ,
AB
W 10J = y
CE
W 20J = ; halle (en J )
CEDC
W .







A) 0 B) 20 C) 10
D) 10 E) 20


86. Un pequeo bloque est conectado al
extremo de un resorte de masa
despreciable, lo separamos
ligeramente de su posicin de
equilibrio una distancia A y en
seguida lo soltamos, de modo que el
bloque se desliza sobre el piso
horizontal liso; entonces es correcto
que:
I. Durante el movimiento del bloque
las fuerzas que realizan trabajo
sobre el son conservativas.
II. La energa cintica es mxima en la
posicin de equilibrio.
III. La grfica energa mecnica versus
posicin es:








A) Solo I B) Todas C) I y II
D) I y III E) II y III

87. Identifique la veracidad (V) o falsedad
(F) de las siguientes proposiciones:
I. Si el trabajo de una fuerza F

por 2
caminos C
1
y C
2
diferentes, entre 2
puntos fijos, es el mismo, se
concluye que F

es conservativa.
II. La energa potencial gravitatoria
requiere de un nivel horizontal de
referencia.
III. La energa potencial elstica de un
resorte de constante k, deformado
x A = , es
2
1
kA
2
.

A) VVV B) FVV C) FVF
D) VVF E) FFF




37
F
m
0,6
0
80
x(m)
1,2
F(N)
B C
E
D
A
A
+A
x
E
M

CICLO PREUNIVERSITARIO ADMISIN -II SEMINARIO N 02

CEPRE-UNI FSICA 18
88. Un cuerpo de masa m se desliza sin
friccin sobre un plano inclinado, si se
suelta desde una altura h
0
respecto al
piso y considerando que h es la
posicin del bloque en su descenso
respecto al piso en un tiempo t 0 > ,
indique cul de los grficos
mostrados representa mejor el trabajo
que realiza el peso del cuerpo en
funcin de h?






A) B)






C) D)






E)

89. A un trineo de kg que se
encuentra sobre un lago congelado se
le imparte una rapidez inicial de 5 m/s.
Si el coeficiente de friccin cintica
entre el trineo y el hielo es 0,1 = ;
cul es la longitud que recorre el
trineo, hasta detenerse?
A) 12,5 B) 7,5 C) 5,5
D) 2,5 E) 1,15

90. El muchacho de la figura tira de la
cuerda con una fuerza cuya magnitud
se incrementa desde cero hasta que l
quede suspendido. Si su masa es de
60 kg y k 500 N/m = , calcule el trabajo
realizado sobre el resorte. (en J )









A) 180 B) 360 C) 540
D) 720 E) 900

91. El bloque de g comprime al
resorte ( ) k 100 N/m = y luego se
suelta. Calcule cul debe ser la
mnima longitud que debe comprimirse
el resorte (en mm) de tal manera que
el bloque pueda recorrer la
circunferencia que se muestra en la
figura.





A) 70 B) 80 C) 100
D) 120 E) 160

92. Una esferita se encuentra suspendida
del extremo libre de un hilo muy largo.
Qu rapidez (en m/s) debe
proporcionarse a la esferita para que
ascienda hasta 1,8 metros?
A) 2 B) 3 C) 4
D) 5 E) 6

93. En el sistema mostrado en la figura
M kg = , m 5 kg = , h 3 m = y la
polea es ideal. Si el sistema parte del
reposo, emplee el principio de la
conservacin de la energa para
encontrar la velocidad con que el
cuerpo M llegar al piso.

W(J )
h(m)
W(J )
h(m)
W(J )
h(m)
W(J )
h(m)
W(J )
h(m)
CICLO PREUNIVERSITARIO ADMISIN -II SEMINARIO N 02

CEPRE-UNI FSICA 19









A) 6 B) 8 C) 6 2
D) 12 E) 14

94. Un bloque de masa m 2kg = parte del
reposo y realiza MRUV con
2
a 0,5 m/s = . Para un tiempo de s
halle la potencia media (en W)
compare con el promedio de las
potencias instantneas, inicial y final,
en dicho intervalo
A) 1,5 B) 2,0 C) 2,5
D) 3,0 E) 3,5

95. Se lanza en A un bloque de 2 kg con
una rapidez inicial de 12 m/s llegando
a B con una rapidez de 4 m/s.
Determine la potencia media
desarrollada por la friccin (en W)
entre x 0 = y x 10 m = .








A) 9,6 B) -12,5 C) -18,0
D) 20,5 E) -22,4

96. Sobre el bloque inicialmente en reposo
se aplica una fuerza horizontal
variables segn indica la grfica.
Cunto trabajo (en J ) hace la fuerza
desde x 0 m = hasta x 10 m = ?
( )
k
0,05 = . Cul es la potencia
instantnea neta (en W) desarrollada
en x 10 m = ?










A) 200 ; 219 B) 400 ; 184
C) 200 ; 184 D) 400 ; 219
E) 100 ; 184

97. Un bloque de kg se desplaza
horizontalmente con una rapidez
constante de 100 m/s sobre una
superficie rugosa (
k
0,8 = ) por accin
de una fuerza horizontal F

. Determine
la potencia (en kW) desarrollada por la
fuerza.
A) 80 B) 64 C) 62
D) 40 E) 16

98. En un edificio se requiere elevar agua
hasta una altura de 10 m, para ello se
usa una bomba de agua cuya
eficiencia es 40%, si la bomba es
capaz de elevar 600 litros de agua en
1 minuto. Calcule la potencia de la
bomba (en kW).
A) 1 B) 2 C) 2,5
D) 6 E) 60
30
A
B
10 m
x
h

M

m
F(N)
x(m)
60
20
10
0
30 kg
k

F

You might also like